Question

In full detail, find the mle of f(x)=e^-(x-theta). theta<=x

In full detail, find the mle of f(x)=e^-(x-theta). theta<=x

Homework Answers

Know the answer?
Your Answer:

Post as a guest

Your Name:

What's your source?

Earn Coins

Coins can be redeemed for fabulous gifts.

Not the answer you're looking for?
Ask your own homework help question
Similar Questions
For x1=2 x2=1.5 and x3=5 data drawn (iid) from f_x(x, theta)= [x^3 exp(-x/theta)]/[6 theta^4] when x>0...
For x1=2 x2=1.5 and x3=5 data drawn (iid) from f_x(x, theta)= [x^3 exp(-x/theta)]/[6 theta^4] when x>0 find the MLE for theta?
Let X1,...,Xn be iid from Poisson(theta), Set k(theta)=exp(-theta). (a) What is the MLE of k(theta)? Is...
Let X1,...,Xn be iid from Poisson(theta), Set k(theta)=exp(-theta). (a) What is the MLE of k(theta)? Is it unbiased? (b) Obtain the CRLB for any unbiased estimator of k(theta).
Find the maximum likelihood estimator (MLE) for the parameter t in the Cauchy probability density f(x;t)=...
Find the maximum likelihood estimator (MLE) for the parameter t in the Cauchy probability density f(x;t)= 1/ [ Pi t(1+(x/t)^2]
Find the maximum likelihood estimator (MLE) for the parameter t in the Cauchy probability density f(x;t)=...
Find the maximum likelihood estimator (MLE) for the parameter t in the Cauchy probability density f(x;t)= 1/ [ Pi t(1+(x/t)^2]
find the derivative of the functions T(theta)=cos(theta^2 + 3theta - 10) f(x)=(3x/(x^2-1))^3/2
find the derivative of the functions T(theta)=cos(theta^2 + 3theta - 10) f(x)=(3x/(x^2-1))^3/2
Suppose X_1, X_2, … X_n is a random sample from a population with density f(x) =...
Suppose X_1, X_2, … X_n is a random sample from a population with density f(x) = (2/theta)*x*e^(-x^2/theta) for x greater or equal to zero. Determine if Theta-Hat_1 (MLE) is a minimum variance unbiased estimator for thet Determine if Theta-Hat_2 (MOM) is a minimum variance unbiased estimator for theta.
f(x,theeta) = theeta x ^(theta-1) find the UMVUE for theeta and 1/theeta
f(x,theeta) = theeta x ^(theta-1) find the UMVUE for theeta and 1/theeta
Suppose that X follows a gamma distribution where alpha=2 but theta is unknown with the following...
Suppose that X follows a gamma distribution where alpha=2 but theta is unknown with the following observed values 50 100 200 300 1000 1) Find the sample mean of X 2) Find the MLE of theta
Y is a random variable with pdf f(y;theta) = ((1- y) ^ theta ) * (theta...
Y is a random variable with pdf f(y;theta) = ((1- y) ^ theta ) * (theta + 1) 0 < y < 0, theta > 0 Find sufficient statistic and UMVUE for theta given E(ln(1 - y)) = - 1/theta + 1
A)  If f (x)= e^cosx find f'(π /2) B) find f'(x) if f(x) =2e^x c) if f(x)=1/e^x...
A)  If f (x)= e^cosx find f'(π /2) B) find f'(x) if f(x) =2e^x c) if f(x)=1/e^x find f''(-1) d)Find  if f(x) =90 (2^x/3) find f'(3) No need to show any work, I just need the answer to each question to check my work. Thanks
ADVERTISEMENT
Need Online Homework Help?

Get Answers For Free
Most questions answered within 1 hours.

Ask a Question
ADVERTISEMENT